Login

Welcome, Guest. Please login or register.

March 28, 2024, 11:48:14 pm

Author Topic: VCE Physics Question Thread!  (Read 603206 times)  Share 

0 Members and 1 Guest are viewing this topic.

speedy

  • Victorian
  • Forum Obsessive
  • ***
  • Posts: 336
  • Respect: 0
  • School Grad Year: 2014
Re: VCE Physics Question Thread!
« Reply #690 on: November 01, 2014, 04:40:43 pm »
0
Not sure if this has been mentioned yet, but VCAA don't provide super detailed solutions (enough to get the full 3 marks) to worded questions because everyone would just copy it down onto their cheat sheet.

Lol I never knew that aha... pretty funny...

Easiest fix ever: remove the bloody cheat sheet and make people actually remember stuff. Like, what, 99% of the other subjects?

I agree with you, but from what I've seen (in my class), if people don't actually understand the content, they do badly, regardless of the fact that they have a full cheat sheet with size 6 font lol.

Also, are incandescent globes on the course? I've been doing pre-2006 exams and in the sections for Light and Matter, it's got questions on incandescent lights and I'm not sure whether or not they're still on the course. :/ thanks!

Production of incoherent light was moved to the photonics detailed study.
Physics [50] | Chemistry [45] | English [42] | IT:SD [44]
ATAR: 98.95

speedy

  • Victorian
  • Forum Obsessive
  • ***
  • Posts: 336
  • Respect: 0
  • School Grad Year: 2014
Re: VCE Physics Question Thread!
« Reply #691 on: November 01, 2014, 11:52:38 pm »
0
When answering Lenz's law questions, do we have to go through ALL of the subsequent effects:

eg. "thus induced EMF will produce a current that flows in a direction which produces a field that opposes this change in flux"

or

"thus a field will be produced that opposes this change in flux"

(This might seem stupid but I usually write the former, I want to know what others write :) )
« Last Edit: November 01, 2014, 11:58:58 pm by speedy »
Physics [50] | Chemistry [45] | English [42] | IT:SD [44]
ATAR: 98.95

PB

  • Victorian
  • Forum Obsessive
  • ***
  • Posts: 387
  • Hi!
  • Respect: +16
  • School: Class of 2018: Griffith BOHDS II
  • School Grad Year: 2013
Re: VCE Physics Question Thread!
« Reply #692 on: November 02, 2014, 01:15:59 am »
0
I usually write the former too. I think its actually better as it follows a logical progression of explanations that makes more sense to me rather than just "a field will be produced that opposes this change in flux".
Also, the former would probably impress the examiner more too as it shows that you know WHY an opposing field is created, not just that it happens because of lenz's law.
« Last Edit: November 02, 2014, 04:01:23 pm by PB »
2013
Physics:50 (Premier's)

ATAR: 99.55

Selling my neat, concise cheatsheet for only $15! Has got extra shortcut formulas which you can get nowhere else, and includes vital watchouts for the exam!

yang_dong

  • Victorian
  • Forum Regular
  • **
  • Posts: 74
  • Respect: 0
  • School: Mac.Robertson Girls' High School
Re: VCE Physics Question Thread!
« Reply #693 on: November 02, 2014, 05:27:50 pm »
0
what's the difference between dynamic loudspeakers and velocity microphones? Cause velocity microphones aren't on the Jacaranda textbook that my school studies off? :(

magneto

  • Victorian
  • Adventurer
  • *
  • Posts: 22
  • Respect: 0
Re: VCE Physics Question Thread!
« Reply #694 on: November 02, 2014, 05:50:49 pm »
0
what mass should i use 33, 44 or 77??
N = mg - ma?

bts

  • Victorian
  • Forum Regular
  • **
  • Posts: 69
  • Respect: 0
Re: VCE Physics Question Thread!
« Reply #695 on: November 02, 2014, 06:06:29 pm »
0
Explain why adding a soft iron core increases the strength of an electromagnet.

please help :)

Zealous

  • ATAR Notes Lecturer
  • Victorian
  • Forum Leader
  • ****
  • Posts: 889
  • zeal: great enthusiasm in pursuit of an objective.
  • Respect: +242
Re: VCE Physics Question Thread!
« Reply #696 on: November 02, 2014, 06:34:35 pm »
0
what mass should i use 33, 44 or 77??
N = mg - ma?
Depends on how you want to approach the question.

Simplest way is to use ma=mg-N with the 33kg mass where N is the force exerted by B on A (working against the gravitational force of A).

So

vce:
2013: Further [50] (+Premier's) | Methods [48]
2014: Physics [50] | Specialist | Accounting | English Language || ATAR: 99.70 + Australian Student Prize!
uni:
2015: Bachelor of Commerce and Engineering (Honours)

lzxnl

  • Victorian
  • ATAR Notes Legend
  • *******
  • Posts: 3432
  • Respect: +215
Re: VCE Physics Question Thread!
« Reply #697 on: November 02, 2014, 07:19:34 pm »
0
Explain why adding a soft iron core increases the strength of an electromagnet.

please help :)

To put it simply, the iron itself is magnetic, so you're adding another magnet on top of an existing magnet.
2012
Mathematical Methods (50) Chinese SL (45~52)

2013
English Language (50) Chemistry (50) Specialist Mathematics (49~54.9) Physics (49) UMEP Physics (96%) ATAR 99.95

2014-2016: University of Melbourne, Bachelor of Science, Diploma in Mathematical Sciences (Applied Maths)

2017-2018: Master of Science (Applied Mathematics)

2019-2024: PhD, MIT (Applied Mathematics)

Accepting students for VCE tutoring in Maths Methods, Specialist Maths and Physics! (and university maths/physics too) PM for more details

Yacoubb

  • Guest
Re: VCE Physics Question Thread!
« Reply #698 on: November 02, 2014, 07:31:32 pm »
0
Explain why adding a soft iron core increases the strength of an electromagnet.

please help :)

I believe that soft iron has high magnetic permeability, and so by adding another magnet, you're increasing the strength of this electromagnetic.

Bestie

  • Victorian
  • Trendsetter
  • **
  • Posts: 130
  • Respect: 0
  • School: Random
Re: VCE Physics Question Thread!
« Reply #699 on: November 02, 2014, 08:46:38 pm »
0
STAV 2013
can someone please help me with question 3,4 and 14 in the attachment? all multiple choice questions
thank you

speedy

  • Victorian
  • Forum Obsessive
  • ***
  • Posts: 336
  • Respect: 0
  • School Grad Year: 2014
Re: VCE Physics Question Thread!
« Reply #700 on: November 02, 2014, 09:13:16 pm »
+1
STAV 2013
can someone please help me with question 3,4 and 14 in the attachment? all multiple choice questions
thank you

These questions are weird as lol...

3) Don't have the image?

4) Bombardment of electrons is off the study design now... This doesn't seem to be a question aligned with the current course, or maybe my knowledge is lacking lol. My guess is A though.

14) Is it D? Although it wouldn't exactly half the current, it is the best way.
Physics [50] | Chemistry [45] | English [42] | IT:SD [44]
ATAR: 98.95

Bestie

  • Victorian
  • Trendsetter
  • **
  • Posts: 130
  • Respect: 0
  • School: Random
Re: VCE Physics Question Thread!
« Reply #701 on: November 03, 2014, 08:36:12 pm »
0
hey speedy! :)

question 3 is above question 4 in the same attachment.

ans to question 4 is B?


speedy

  • Victorian
  • Forum Obsessive
  • ***
  • Posts: 336
  • Respect: 0
  • School Grad Year: 2014
Re: VCE Physics Question Thread!
« Reply #702 on: November 03, 2014, 10:05:28 pm »
0
hey speedy! :)

question 3 is above question 4 in the same attachment.

ans to question 4 is B?

As in there is no diagram.

Yeah, maybe someone else could help, but bombardment with electrons has been removed from the study design so it's not something you're meant to know.
Physics [50] | Chemistry [45] | English [42] | IT:SD [44]
ATAR: 98.95

rui97

  • Victorian
  • Trailblazer
  • *
  • Posts: 32
  • Respect: 0
  • School: The University of Melbourne
  • School Grad Year: 2018
Re: VCE Physics Question Thread!
« Reply #703 on: November 05, 2014, 11:05:02 pm »
0
hey guys
for significant diffraction to occur, does lambda/slit width have to be greater than or equal to one or does it need to have the same order of magnitude? I havent been able to get a definite answer, what do you guys think?
2013: Further Mathematics [50] (Premier's Award)
2014: Mathematical Methods [50] | Specialist Mathematics [49] | Chemistry [45] | Physics [45] | English Language [38]
2015: Bachelor of Commerce - Actuarial Studies

lzxnl

  • Victorian
  • ATAR Notes Legend
  • *******
  • Posts: 3432
  • Respect: +215
Re: VCE Physics Question Thread!
« Reply #704 on: November 06, 2014, 09:17:59 am »
+1
hey guys
for significant diffraction to occur, does lambda/slit width have to be greater than or equal to one or does it need to have the same order of magnitude? I havent been able to get a definite answer, what do you guys think?

For noticeable diffraction, they need to be of the same order of magnitude. For complete diffraction, you need your slit to be smaller than the wavelength.
2012
Mathematical Methods (50) Chinese SL (45~52)

2013
English Language (50) Chemistry (50) Specialist Mathematics (49~54.9) Physics (49) UMEP Physics (96%) ATAR 99.95

2014-2016: University of Melbourne, Bachelor of Science, Diploma in Mathematical Sciences (Applied Maths)

2017-2018: Master of Science (Applied Mathematics)

2019-2024: PhD, MIT (Applied Mathematics)

Accepting students for VCE tutoring in Maths Methods, Specialist Maths and Physics! (and university maths/physics too) PM for more details